0 Daumen
1,2k Aufrufe

der Grenzwert folgender Reihe soll bestimmt werden:

∑ (2^n / ( 5^n - 1))

n startet bei 0 und läuft gegen unendlich. Meine Frage ist nun, wie kann ich das in eine geometrische Reihe umformen? Wenn die -1 im Zähler stünde, wäre das ja einfach.

Avatar von

Kleine Korrektur: n startet natürlich nicht bei 0, sondern bei 1. Aber es geht ja auch nur um den richtigen Ansatz.

Die Konvergenz kann man ja mittels Quotientenkriterium leicht nachweisen....

2 Antworten

0 Daumen

2^n/(5^n-1) = (2^n-1+1)/(5^n-1) = (2^n-1)/(5^n-1) + 1/(5^n-1)

Avatar von 81 k 🚀

Aber inwiefern hilft mir diese Umformung weiter? Für eine geometrische Reihe brauche ich ja Summenden in der Form a*b^n , und die kann ich doch nur bilden, wenn im Nenner keine Summe bzw. Differenz mehr steht. Oder sehe ich das falsch?

Oder sehe ich das falsch?

Das siehst Du im Prinzip schon richtig! Zunächst mal bilden die Elemente \(a_n\) der obige Reihe keine geometrische Folge, da der Quotient \(a_{n+1}/a_n\) nicht konstant ist. Evt. könnte man sich vorstellen, dass eine geometrische Reihe existiert, die auf den selben Grenzwert zuläuft.

Bist Du sicher, dass man den Grenzwert über eine geometrische Reihe berechnen kann? Steht das in der Aufgabenstellung?

Nein, ganz sicher bin ich nicht, aber bisher hatten wir immer nur Aufgaben, die auf geometrische Reihen hinausliefen, und haben auch auch nur das gelernt bisher. Wie kann man den Grenzwert denn sonst berechnen in diesem Fall?

0 Daumen

mir ist kein Verfahren bekannt, mit dem Du den Wert dieser Reihe exakt bzw. algebraisch berechnen kannst. Du kannst den Wert aber sehr gut annähern. Wähle dazu einen Index \(k \gt 1\), \(k \in \mathbb{N}\). Dann ist sicher $$\sum_{n=1}^{k-1} \frac {2^n}{5^n-1} \lt \sum_{n=1}^{\infty} \frac {2^n}{5^n-1} \lt \sum_{n=1}^{k-1} \frac {2^n}{5^n-1} \space + \sum_{n=k}^{\infty} \frac {2^n}{4^n}$$Den linken Term kann man ausrechnen, da \(k\) endlich ist. Den zweiten Teil des rechten Term kann man noch etwas umformen:$$\sum_{n=k}^{\infty} \frac {2^n}{4^n} = \frac 1{2^k} \sum_{j=0}^{\infty} \left( \frac 12 \right)^j = \frac 1{2^{k-1}}$$und z.B. für \(k=20\) ergibt sich dann die Ungleichung (Werte auf 7 Stellen gerundet)$$0,7738940 \lt \sum_{n=1}^{\infty} \frac {2^n}{5^n-1} \lt 0,7738940 + 0,0000019 = 0,7738959$$und damit eine gute Näherung auf mindestens 5 Dezimalstellen hinter dem Komma.

Avatar von 48 k

Ein anderes Problem?

Stell deine Frage

Willkommen bei der Mathelounge! Stell deine Frage einfach und kostenlos

x
Made by a lovely community